LSAT and Law School Admissions Forum

Get expert LSAT preparation and law school admissions advice from PowerScore Test Preparation.

General questions relating to the LSAT or LSAT preparation.
 brittany1990
  • Posts: 21
  • Joined: Jul 11, 2012
|
#5981
Hi!

I wasn't sure how to diagram the conditional reasoning for this question so if you could check below to see if it's right that would be great!

P: not increase enrollment :arrow: reduce spending
P: increase enrollment :arrow: marketing
C: maintain quality :arrow: marketing


Also, I realized that I needed to connect reduce spending and maintain quality so I picked D. However, I'm not really sure how this answer choice would fit into the conditional reasoning I diagrammed above.

Thanks!
Brittany
 Nikki Siclunov
PowerScore Staff
  • PowerScore Staff
  • Posts: 1362
  • Joined: Aug 02, 2011
|
#5996
Hi Brittany,

Your analysis of the conditional reasoning in this argument is correct. The structure is as follows:

Premise: NOT increase enrollment :arrow: reduce spending
Premise: Increase enrollment :arrow: marketing

Conclusion: maintain quality :arrow: marketing

To figure out the exact nature of the conditional connection required, connect the premises as follows:

Premise 1+2: NOT reduce spending :arrow: increase enrollment :arrow: marketing

To justify the conclusion (maintain quality :arrow: marketing) we need to connect the rogue element ("maintain quality") to either of the two conditions for which "marketing" is a necessary condition. Thus, we should be looking for one of the following two statements:

maintain quality :arrow: NOT reduce spending
maintain quality :arrow: increase enrollment

Either of these statements (or their contrapositives) would justify the conclusion, proving that answer choice (D) is correct:

maintain quality :arrow: NOT reduce spending :arrow: increase enrollment :arrow: marketing



Let me know if you have any questions!
 Jon Denning
PowerScore Staff
  • PowerScore Staff
  • Posts: 904
  • Joined: Apr 11, 2011
|
#5998
Hey Brittany - I see Nikki responded as well, but since I already wrote up an answer here I figured I'd post mine too :)

Tricky Justify question here. First off, congrats on the diagrams--they all look correct to me. The trick then becomes how to use an answer to prove that maintaining quality requires they market more aggressively.

Let's look at D and combine it with the premises to see how they're all connected. D says essentially Reduce Spending --> NOT Maintain Quality. Contrapositive: Maintain Quality --> NOT Reduce Spending

Look at how that connects to the contrapositive of your first premise (Not reduce spending --> Increase enrollment):

Maintain Quality --> NOT Reduce Spending --> Increase Enrollment

Which of course connects to your second premise (Increase enrollment --> Marketing):

Maintain Quality --> NOT Reduce Spending --> Increase Enrollment --> Marketing

Looking at the chain above, what we've just done is prove the connection between quality and marketing, where Maintain Quality --> Marketing. That's the conclusion, so D justifies it.

Two more things about this one I found interesting. One, we see a new term in the conclusion--maintain quality of education--which means that term MUST also be in the correct answer choice. Sure enough, D has that same idea (looks negated, but it's just from the contrapositive construction).

Secondly, this structure isn't all that uncommon for justify. Consider this generic, simplified example:

Prem 1: A --> B
Conclusion: A --> C

How can I prove that? Add B --> C as another premise to get A --> B --> C, which proves the conclusion A --> C.

Again, simplified, but the conditional linkage idea is more or less the same as this stimulus.

Hope that helps!

Get the most out of your LSAT Prep Plus subscription.

Analyze and track your performance with our Testing and Analytics Package.